Column

Consider a 5 m long column with a modulus of elasticity of 10 GPa and a moment of inertia of 400 cm^4 is subjected to buckling. If the column has a fixed-fixed connection, what is the critical axial load in kN?

Expand Hint
Euler’s formula:
$$$P_{cr}=\frac{\pi^2EI}{(Kl)^2}$$$
where $$P_{cr}$$ is the critical axial load for long columns subjected to buckling, $$E$$ is the elastic modulus, $$I$$ is the moment of inertia, $$K$$ is the effective length factor to account for end supports, and $$l$$ is the unbraced column length.
Hint 2
Theoretical effective length factors for columns include:
  • Pinned-pinned - $$K=1.0$$
  • Fixed-fixed - $$K=0.5$$
  • Fixed-pinned - $$K=0.7$$
  • Fixed-free - $$K=2.0$$
Euler’s formula:
$$$P_{cr}=\frac{\pi^2EI}{(Kl)^2}$$$
where $$P_{cr}$$ is the critical axial load for long columns subjected to buckling, $$E$$ is the elastic modulus, $$I$$ is the moment of inertia, $$K$$ is the effective length factor to account for end supports, and $$l$$ is the unbraced column length. Theoretical effective length factors for columns include:
  • Pinned-pinned - $$K=1.0$$
  • Fixed-fixed - $$K=0.5$$
  • Fixed-pinned - $$K=0.7$$
  • Fixed-free - $$K=2.0$$

Thus,
$$$P_{cr}=\frac{\pi^2(10\cdot 10^9Pa)(4\cdot 10^{-6}m^4)}{(0.5\cdot 5m)^2}=\frac{(\pi^2)40\cdot 10^3N\cdot m^4}{6.25m^2(m^2)}$$$
$$$=\frac{394,784.2N}{6.25}=63,165.5N=63\:kN$$$
63 kN
Time Analysis See how quickly you looked at the hint, solution, and answer. This is important for making sure you will finish the FE Exam in time.
  • Hint: Not clicked
  • Solution: Not clicked
  • Answer: Not clicked


Similar Problems from FE Section: Columns
086. Euler's Formula
207. Euler's Formula for Columns
413. A Column
418. Buckling Stress
563. Column Area
567. Column Modulus